Zur Vertretung der Lorentz-Gruppe [geschlossen]

Ich gehe die Notizen zu QFT von Srednicki durch .

Bei der Beschreibung von Fermionen führt er von Anfang an die Lorentzgruppe und ihre Algebra ein und beweist, dass sie äquivalent zu zwei Kopien von ist S U ( 2 ) , so dass eine Darstellung beispielsweise durch zwei (halbe) ganze Zahlen angegeben wird N , N ' (siehe Seite 213-214). Er schreibt eine solche Darstellung wie ( 2 N + 1 , 2 N ' + 1 ) .

Beispielsweise sind einige wichtige Darstellungen ( 1 , 1 ) : Skalar, ( 2 , 1 ) : linkshändiger Spinor und so weiter. Einige Seiten später (S. 217) schreibt er die Beziehung 2 2 = 1 3 , was nur das übliche Ergebnis der Addition von Drehimpuls ist. Mein Problem ist, einige Seiten später (S. 219) schreibt er die folgende "gruppentheoretische Beziehung"

( 2 , 2 ) ( 2 , 2 ) = ( 1 , 1 ) ( 1 , 3 ) ( 3 , 1 ) ( 3 , 3 ) .
Es fällt mir schwer, diesen Zusammenhang zu verstehen.

Auf den ersten Blick sieht es so aus, als müssten wir vier Spin-Halb-Impulse hinzufügen, d.h.

( 2 , 2 ) ( 2 , 2 ) = 2 2 2 2.
Wenn ich die üblichen Schritte durchführe, um eine solche Summe zu konstruieren, erhalte ich auf keinen Fall das erwartete Ergebnis.

Andererseits, wenn ich schreibe ( 2 , 2 ) = 1 3 und „verteilen über „Als ob es sich um tatsächliche Produkte und Summen handelte, erhalte ich das von Srednicki angegebene Ergebnis, aber ich habe das Gefühl, dass etwas daran nicht stimmt. Vielleicht habe ich das Gefühl, dass es falsch ist, nur weil mir etwas fehlt oder ich es nicht verstehe.

Wenn diese „verteilen über " ist das Richtige, ich würde mich sehr freuen, wenn jemand erklärt, warum das so ist. Wenn es nicht das Richtige ist, dann würde ich mich freuen, wenn mir jemand sagt, wie ich mit solchen "gruppentheoretischen Beziehungen" umgehen soll, oder wo finde ich literatur zu diesem thema.

Zum Beispiel schreibt Srednicki auf Seite 218

( 2 , 1 ) ( 1 , 2 ) ( 2 , 2 ) = ( 1 , 1 ) . . .

Wenn mein Ansatz der richtige ist, dann als

( 2 , 1 ) ( 1 , 2 ) = 1 3 = ( 2 , 2 ) ,
Die vollständige Antwort ist
( 2 , 1 ) ( 1 , 2 ) ( 2 , 2 ) = ( 2 , 2 ) ( 2 , 2 ) = ( 1 , 1 ) ( 1 , 3 ) ( 3 , 1 ) ( 3 , 3 ) .
Ist es das?

Vielleicht sollte ich hinzufügen, dass ich keine Probleme habe, wenn ich mit einzelnen Darstellungen umgehe (falls das Sinn macht): Ich verstehe, warum (1,1) eine skalare Darstellung ist, ihre Eigenschaften und so weiter. Außerdem weiß ich, wie man Darstellungen tensoriert (z ( 3 , 2 ) = 4 1 , wobei der Isomorphismus beispielsweise durch die Clebsch-Gordan-Koeffizienten gegeben ist). Mein Problem ist der Umgang mit Ausdrücken wie ( M 1 , N 1 ) ( M 2 , N 2 ) . . . = ( A , B ) ( C , D ) . . . ; Ich habe das Gefühl, dass es viele mögliche Ergebnisse gibt, die alle durch Isomorphismen zusammenhängen (ist das richtig?).
Deine Notation ergibt für mich keinen Sinn. Die skalare Wiederholung sollte sein ( 0 , 0 ) , nicht ( 1 , 1 ) , da es sich um die Zero-Spin- Darstellung handelt. Warum gibt es Gleichungen, bei denen ein Vertreter zwei Bezeichnungen hat, wie z ( 2 , 1 ) , und noch ein einziger?! Zum Schluss, was ist Ihre eigentliche Frage (außer „Ist das richtig“)?
Ich folge der Notation von Srednicki, wo eine Darstellung geschrieben wird als ( 2 N + 1 , 2 N ' + 1 ) , also der Skalar ( N = 0 , N ' = 0 ) Ist ( 1 , 1 ) . Außerdem, so wie ich es verstehe, ( A , B ) = A B (Vielleicht sollte ich schreiben statt =, da es sich um einen Isomorphismus handelt, richtig?. So ( 2 , 1 ) = 2 1 und so weiter (deshalb schreibe ich manchmal ( A , B ) und manchmal einzelne Nummern. Schließlich bezieht sich meine Frage darauf, wie man die von Srednicki angegebene Beziehung beweist und analoge Ausdrücke berechnet. Als alle für Ihre Zeit :)
Diese Frage (v2) scheint ein Archetyp eines mathematischen Problems zu sein, das in vielen Bereichen der Physik vorkommt, zB QCD, und das die Community konsequent nicht nach Math.SE migrieren möchte, vgl. diesen Metapost .
Rezensenten stimmen jedoch dafür, es auf Math.SE zu migrieren. Ich schließe diese Frage als Hausaufgaben-ähnliche Frage, teilweise um zu verhindern, dass sie zu Math.SE migriert wird.

Antworten (1)

Es gibt einen feinen Unterschied zwischen sagen ( 2 , 2 ) Und 2 2 . Im letzteren Fall stellen wir uns vor, dass sich beide Wiederholungen unter demselben Element der Gruppe transformieren S U ( 2 ) . Im ersteren Fall denken wir an ( 2 , 2 ) als Transformation unter der Lorentz-Gruppe, die zwei unterschiedliche Kopien von enthält S U ( 2 ) . Nennen Sie eine Kopie der L kopieren und die andere die R Kopieren. Dann werden die vier Basisvektoren von ( 2 , 2 ) Sind 0 L 0 R , 0 L 1 R , usw. Diese vier Basisvektoren trennen sich nicht in 1 3 da ich Elemente der Lorentzgruppe auswählen kann, die nur eine der beiden Darstellungen drehen.

Also denken Sie an ( 2 , 2 ) ( 2 , 2 ) = ( 2 2 , 2 2 ) , die Basisvektoren wie zB hat 0 L 1 1 R 1 0 L 2 1 R 2 , also kann ich die Addition von Drehimpulsen zwischen den beiden Ls und Rs anwenden. Dann ( 1 3 , 1 3 ) bedeutet, dass Sie alle Basisvektoren von nehmen ( 1 3 ) L und Tensorprodukt mit allen Basisvektoren von ( 1 3 ) R . Es verteilt sich also.

Also beim Schreiben ( 2 , 1 ) ( 1 , 2 ) stell es dir vor wie

( 2 , 1 ) ( 1 , 2 ) = ( 2 1 , 1 2 ) = ( 2 , 2 )

Nun, das war wirklich hilfreich :) Also beim Umgang mit größeren Ausdrücken wie ( 1 , 2 ) ( 2 , 3 ) ( 1 , 3 ) , wir müssen eigentlich die drei Impulse hinzufügen, richtig? Ich meine, wir haben = ( 1 2 1 , 2 3 3 ) , und wir müssen rechnen 1 2 1 usw.
Ja, du hast es.